K
Khách

Hãy nhập câu hỏi của bạn vào đây, nếu là tài khoản VIP, bạn sẽ được ưu tiên trả lời.

13 tháng 3 2018

Áp dụng bất đẳng thức Cauchy-Schwarz: \(\frac{a^2}{b+c}+\frac{b^2}{a+c}+\frac{c^2}{a+b}\ge\frac{\left(a+b+c\right)^2}{2\left(a+b+c\right)}=\frac{a+b+c}{2}\)

13 tháng 3 2018

áp dụng bất đẳng thức Cauchy-Schwarz: \(\frac{a^2}{b+c}+\frac{b^2}{a+c}+\frac{c^2}{a+b}\ge\frac{\left(a+b+c\right)^2}{2\left(a+b+c\right)}=\frac{a+b+c}{2}\)

9 tháng 8 2019

2) Theo nguyên lí Dirichlet, trong ba số \(a^2-1;b^2-1;c^2-1\) có ít nhất hai số nằm cùng phía với 1.

Giả sử đó là a2 - 1 và b2 - 1. Khi đó \(\left(a^2-1\right)\left(b^2-1\right)\ge0\Leftrightarrow a^2b^2-a^2-b^2+1\ge0\)

\(\Rightarrow a^2b^2+3a^2+3b^2+9\ge4a^2+4b^2+8\)

\(\Rightarrow\left(a^2+3\right)\left(b^2+3\right)\ge4\left(a^2+b^2+2\right)\)

\(\Rightarrow\left(a^2+3\right)\left(b^2+3\right)\left(c^2+3\right)\ge4\left(a^2+b^2+1+1\right)\left(1+1+c^2+1\right)\) (2)

Mà \(4\left[\left(a^2+b^2+1+1\right)\left(1+1+c^2+1\right)\right]\ge4\left(a+b+c+1\right)^2\) (3)(Áp dụng Bunhicopxki và cái ngoặc vuông)

Từ (2) và (3) ta có đpcm.

Sai thì chịu

9 tháng 8 2019

Xí quên bài 2 b:v

b) Không mất tính tổng quát, giả sử \(\left(a^2-\frac{1}{4}\right)\left(b^2-\frac{1}{4}\right)\ge0\)

Suy ra \(a^2b^2-\frac{1}{4}a^2-\frac{1}{4}b^2+\frac{1}{16}\ge0\)

\(\Rightarrow a^2b^2+a^2+b^2+1\ge\frac{5}{4}a^2+\frac{5}{4}b^2+\frac{15}{16}\)

Hay \(\left(a^2+1\right)\left(b^2+1\right)\ge\frac{5}{4}\left(a^2+b^2+\frac{3}{4}\right)\)

Suy ra \(\left(a^2+1\right)\left(b^2+1\right)\left(c^2+1\right)\ge\frac{5}{4}\left(a^2+b^2+\frac{1}{4}+\frac{1}{2}\right)\left(\frac{1}{4}+\frac{1}{4}+c^2+\frac{1}{2}\right)\)

\(\ge\frac{5}{4}\left(\frac{1}{2}a+\frac{1}{2}b+\frac{1}{2}c+\frac{1}{2}\right)^2=\frac{5}{16}\left(a+b+c+1\right)^2\) (Bunhiacopxki) (đpcm)

Đẳng thức xảy ra khi \(a=b=c=\frac{1}{2}\)

1 tháng 8 2020

Xét \(\frac{a^3}{a^2+ab+b^2}-\frac{b^3}{a^2+ab+b^2}=\frac{\left(a-b\right)\left(a^2+ab+b^2\right)}{a^2+ab+b^2}=a-b\)

Tương tự, ta được: \(\frac{b^3}{b^2+bc+c^2}-\frac{c^3}{b^2+bc+c^2}=b-c\)\(\frac{c^3}{c^2+ca+a^2}-\frac{a^3}{c^2+ca+a^2}=c-a\)

Cộng theo vế của 3 đẳng thức trên, ta được: \(\left(\frac{a^3}{a^2+ab+b^2}+\frac{b^3}{b^2+bc+c^2}+\frac{c^3}{c^2+ca+a^2}\right)\)\(-\left(\frac{b^3}{a^2+ab+b^2}+\frac{c^3}{b^2+bc+c^2}+\frac{a^3}{c^2+ca+a^2}\right)=0\)

\(\Rightarrow\frac{a^3}{a^2+ab+b^2}+\frac{b^3}{b^2+bc+c^2}+\frac{c^3}{c^2+ca+a^2}\)\(=\frac{b^3}{a^2+ab+b^2}+\frac{c^3}{b^2+bc+c^2}+\frac{a^3}{c^2+ca+a^2}\)

Ta đi chứng minh BĐT phụ sau: \(a^2-ab+b^2\ge\frac{1}{3}\left(a^2+ab+b^2\right)\)(*)

Thật vậy: (*)\(\Leftrightarrow\frac{2}{3}\left(a-b\right)^2\ge0\)*đúng*

\(\Rightarrow2LHS=\Sigma_{cyc}\frac{a^3+b^3}{a^2+ab+b^2}=\Sigma_{cyc}\text{ }\frac{\left(a+b\right)\left(a^2-ab+b^2\right)}{a^2+ab+b^2}\)\(\ge\Sigma_{cyc}\text{ }\frac{\frac{1}{3}\left(a+b\right)\left(a^2+ab+b^2\right)}{a^2+ab+b^2}=\frac{1}{3}\text{​​}\Sigma_{cyc}\left[\left(a+b\right)\right]=\frac{2\left(a+b+c\right)}{3}\)

\(\Rightarrow LHS\ge\frac{a+b+c}{3}=RHS\)(Q.E.D)

Đẳng thức xảy ra khi a = b = c

P/S: Có thể dùng BĐT phụ ở câu 3a để chứng minhxD:

27 tháng 7 2020

1) ta chứng minh được \(\Sigma\frac{a^4}{\left(a+b\right)\left(a^2+b^2\right)}=\Sigma\frac{b^4}{\left(a+b\right)\left(a^2+b^2\right)}\)

\(VT=\frac{1}{2}\Sigma\frac{a^4+b^4}{\left(a+b\right)\left(a^2+b^2\right)}\ge\frac{1}{4}\Sigma\frac{a^2+b^2}{a+b}\ge\frac{1}{8}\Sigma\left(a+b\right)=\frac{a+b+c+d}{4}\)

bài 2 xem có ghi nhầm ko

2 tháng 12 2020

Đặt \(\left(\frac{1}{a},\frac{1}{b},\frac{1}{c}\right)=\left(x,y,z\right)\)

\(x+y+z\ge\frac{x^2+2xy}{2x+y}+\frac{y^2+2yz}{2y+z}+\frac{z^2+2zx}{2z+x}\)

\(\Leftrightarrow x+y+z\ge\frac{3xy}{2x+y}+\frac{3yz}{2y+z}+\frac{3zx}{2z+x}\)

\(\frac{3xy}{2x+y}\le\frac{3}{9}xy\left(\frac{1}{x}+\frac{1}{x}+\frac{1}{y}\right)=\frac{1}{3}\left(x+2y\right)\)

\(\Rightarrow\Sigma_{cyc}\frac{3xy}{2x+y}\le\frac{1}{3}\left[\left(x+2y\right)+\left(y+2z\right)+\left(z+2x\right)\right]=x+y+z\)

Dấu "=" xảy ra khi x=y=z

28 tháng 5 2019

:v~ cả lớp 7 

28 tháng 5 2019

Quá dài dòng ~.~

Có: \(\frac{a}{b}+\frac{b}{c}+\frac{c}{a}=\frac{a^4}{a^3b}+\frac{b^4}{b^3c}+\frac{c^4}{c^3a}\ge\frac{\left(a^2+b^2+c^2\right)^2}{a^3b+b^3c+c^3a}=\frac{9\left(a^2+b^2+c^2\right)^2}{9\left(a^3b+b^3c+c^3a\right)}\)

Cần CM Bđt:

\(\left(a+b+c\right)^2\left(a^2+b^2+c^2\right)\ge9\left(a^3b+b^3c+c^3a\right)\)

hay: \(\left(a^2+b^2+c^2\right)^2+2\left(ab+bc+ac\right)\left(a^2+b^2+c^2\right)\ge9\left(a^3b+b^3c+c^3a\right)\)

Sử dụng Bđt phụ: \(\left(a^2+b^2+c^2\right)^2\ge3\left(a^3b+b^3c+c^3a\right)\)

Thu gọn bất đẳng thức cần CM còn: \(\left(ab+bc+ac\right)\left(a^2+b^2+c^2\right)\ge3\left(a^3b+b^3c+c^3a\right)\)

Cm tương đương là xong.

Như vậy: \(VT\ge\frac{9\left(a^2+b^2+c^2\right)^2}{9\left(a^3b+b^3c+c^3a\right)}\ge\frac{9\left(a^2+b^2+c^2\right)^2}{\left(a+b+c\right)^2\left(a^2+b^2+c^2\right)}=VP\)

End./.

Giúp mình với! Mình đang cần gấp. Các bạn làm được bài nào thì giúp đỡ mình nhé! Cảm ơn!Bài 1: Cho các số thực dương a,b,c. Chứng minh rằng:\(\frac{a^2}{\sqrt{\left(2a^2+b^2\right)\left(2a^2+c^2\right)}}+\frac{b^2}{\sqrt{\left(2b^2+c^2\right)\left(2b^2+a^2\right)}}+\frac{c^2}{\sqrt{\left(2c^2+a^2\right)\left(2c^2+b^2\right)}}\le1\).Bài 2: Cho các số thực dương a,b,c,d. Chứng minh...
Đọc tiếp

Giúp mình với! Mình đang cần gấp. Các bạn làm được bài nào thì giúp đỡ mình nhé! Cảm ơn!

Bài 1: Cho các số thực dương a,b,c. Chứng minh rằng:

\(\frac{a^2}{\sqrt{\left(2a^2+b^2\right)\left(2a^2+c^2\right)}}+\frac{b^2}{\sqrt{\left(2b^2+c^2\right)\left(2b^2+a^2\right)}}+\frac{c^2}{\sqrt{\left(2c^2+a^2\right)\left(2c^2+b^2\right)}}\le1\).

Bài 2: Cho các số thực dương a,b,c,d. Chứng minh rằng:

\(\frac{a-b}{a+2b+c}+\frac{b-c}{b+2c+d}+\frac{c-d}{c+2d+a}+\frac{d-a}{d+2a+b}\ge0\).

Bài 3: Cho các số thực dương a,b,c. Chứng minh rằng:

\(\frac{\sqrt{b+c}}{a}+\frac{\sqrt{c+a}}{b}+\frac{\sqrt{a+b}}{c}\ge\frac{4\left(a+b+c\right)}{\sqrt{\left(a+b\right)\left(b+c\right)\left(c+a\right)}}\).

Bài 4:Cho a,b,c>0, a+b+c=3. Chứng minh rằng: 

a)\(\frac{a^3}{a^2+ab+b^2}+\frac{b^3}{b^2+bc+c^2}+\frac{c^3}{c^2+ca+a^2}\ge1\).

b)\(\frac{a^3}{a^2+b^2}+\frac{b^3}{b^2+c^2}+\frac{c^3}{c^2+a^2}\ge\frac{3}{2}\).

c)\(\frac{a+1}{b^2+1}+\frac{b+1}{c^2+1}+\frac{c+1}{a^2+1}\ge3\).

Bài 5: Cho a,b,c >0. Chứng minh rằng:

\(\frac{2a^2+ab}{\left(b+c+\sqrt{ca}\right)^2}+\frac{2b^2+bc}{\left(c+a+\sqrt{ab}\right)^2}+\frac{2c^2+ca}{\left(a+b+\sqrt{bc}\right)^2}\ge1\).

8
21 tháng 10 2019

1) Áp dụng bunhiacopxki ta được \(\sqrt{\left(2a^2+b^2\right)\left(2a^2+c^2\right)}\ge\sqrt{\left(2a^2+bc\right)^2}=2a^2+bc\), tương tự với các mẫu ta được vế trái \(\le\frac{a^2}{2a^2+bc}+\frac{b^2}{2b^2+ac}+\frac{c^2}{2c^2+ab}\le1< =>\)\(1-\frac{bc}{2a^2+bc}+1-\frac{ac}{2b^2+ac}+1-\frac{ab}{2c^2+ab}\le2< =>\)

\(\frac{bc}{2a^2+bc}+\frac{ac}{2b^2+ac}+\frac{ab}{2c^2+ab}\ge1\)<=> \(\frac{b^2c^2}{2a^2bc+b^2c^2}+\frac{a^2c^2}{2b^2ac+a^2c^2}+\frac{a^2b^2}{2c^2ab+a^2b^2}\ge1\)  (1) 

áp dụng (x2 +y2 +z2)(m2+n2+p2\(\ge\left(xm+yn+zp\right)^2\)

(2a2bc +b2c2 + 2b2ac+a2c2 + 2c2ab+a2b2). VT\(\ge\left(bc+ca+ab\right)^2\)   <=> (ab+bc+ca)2. VT \(\ge\left(ab+bc+ca\right)^2< =>VT\ge1\)  ( vậy (1) đúng)

dấu '=' khi a=b=c

21 tháng 10 2019

4b, \(\frac{a^3}{a^2+b^2}+\frac{b^3}{b^2+c^2}+\frac{c^3}{c^2+a^2}=1-\frac{ab^2}{a^2+b^2}+1-\frac{bc^2}{b^2+c^2}+1-\frac{ca^2}{a^2+c^2}\)

\(\ge3-\frac{ab^2}{2ab}-\frac{bc^2}{2bc}-\frac{ca^2}{2ac}=3-\frac{\left(a+b+c\right)}{2}=\frac{3}{2}\)

2 tháng 7 2019

Áp dụng Svac

\(\Sigma\frac{a^3}{b+c}=\Sigma\frac{a^4}{ab+ac}\ge\frac{\left(a^2+b^2+c^2\right)^2}{2\left(ab+bc+ca\right)}\ge\frac{\left(a^2+b^2+c^2\right)^2}{2\left(a^2+b^2+c^2\right)}=\frac{1}{2}\left(a^2+b^2+c^2\right)\)

"=" tại a=b=c

4 tháng 7 2019

E thử làm cách khác ạ:))

Không mất tính tổng quát,giả sử \(a\ge b\ge c\)

\(\Rightarrow\hept{\begin{cases}a^2\ge b^2\ge c^2\\\frac{a}{b+c}\ge\frac{b}{a+c}\ge\frac{c}{a+b}\end{cases}}\)

Áp dụng BĐT Trebysev ta có:

\(a^2\cdot\frac{a}{b+c}+b^2\cdot\frac{b}{a+c}+c^2\cdot\frac{c}{a+b}\ge\frac{a^2+b^2+c^2}{3}\cdot\left(\frac{a}{b+c}+\frac{b}{c+a}+\frac{c}{a+b}\right)\)

\(\ge\frac{a^2+b^2+c^2}{3}\cdot\frac{3}{2}\left(nesbitt\right)\)

\(=\frac{a^2+b^2+c^2}{2}\)

Dấu "=" xảy ra khi  \(a=b=c\)